- Wed Jun 08, 2016 4:40 pm
#26280
Complete Question Explanation
(The complete setup for this game can be found here: lsat/viewtopic.php?t=10929)
The correct answer choice is (D)
To answer this question correctly, proceed by the process of elimination: any answer choice suggesting a possible solution would be incorrect.
Answer choice (A) is incorrect, because a newsletter with a single F feature and no I or M features can still comply with the rules of the game. Each of the remaining two (or more) features would simply be T’s. Since an F or a T feature would also occupy slot 1, such a solution would not be in violation of the second rule.
Answer choice (B) is incorrect, because a newsletter with a single I feature and no F or M features does not violate any of the rules. As with answer choice (A), each of the remaining two (or more) features would simply be T’s. As long as a T feature occupies slot 1, the solution would comply with the second rule as well:
Answer choice (C) is incorrect for the same reason as answer choice (B) is incorrect. F and T are functionally identical variables, as there are no rules that apply to F but not T (or vice versa). So, if we can have a newsletter with only I and T features, we can also have one with only I and F features. This fact alone would be sufficient to eliminate both answer choices (B) and (C), as the correct answer choice must be logically unique.
Answer choice (D) is the correct answer choice. If there is exactly one M feature and no F or T features, then the maximum number of features in this newsletter would be two. This is because the newsletter can have at most one I feature in accordance with the third rule. Regardless of how these features are spread out, such a solution would violate the fundamental requirement of the scenario that there be at least three features in any issue of the newsletter.
Answer choice (E) is incorrect, because a newsletter with a single M feature and no I or T features can still comply with the rules of the game. Each of the remaining two (or more) features would need to be F’s. As long as one of the F features occupies slot 1, the solution would not be in violation of the second rule:
(The complete setup for this game can be found here: lsat/viewtopic.php?t=10929)
The correct answer choice is (D)
To answer this question correctly, proceed by the process of elimination: any answer choice suggesting a possible solution would be incorrect.
Answer choice (A) is incorrect, because a newsletter with a single F feature and no I or M features can still comply with the rules of the game. Each of the remaining two (or more) features would simply be T’s. Since an F or a T feature would also occupy slot 1, such a solution would not be in violation of the second rule.
Answer choice (B) is incorrect, because a newsletter with a single I feature and no F or M features does not violate any of the rules. As with answer choice (A), each of the remaining two (or more) features would simply be T’s. As long as a T feature occupies slot 1, the solution would comply with the second rule as well:
Answer choice (C) is incorrect for the same reason as answer choice (B) is incorrect. F and T are functionally identical variables, as there are no rules that apply to F but not T (or vice versa). So, if we can have a newsletter with only I and T features, we can also have one with only I and F features. This fact alone would be sufficient to eliminate both answer choices (B) and (C), as the correct answer choice must be logically unique.
Answer choice (D) is the correct answer choice. If there is exactly one M feature and no F or T features, then the maximum number of features in this newsletter would be two. This is because the newsletter can have at most one I feature in accordance with the third rule. Regardless of how these features are spread out, such a solution would violate the fundamental requirement of the scenario that there be at least three features in any issue of the newsletter.
Answer choice (E) is incorrect, because a newsletter with a single M feature and no I or T features can still comply with the rules of the game. Each of the remaining two (or more) features would need to be F’s. As long as one of the F features occupies slot 1, the solution would not be in violation of the second rule:
You do not have the required permissions to view the files attached to this post.